Diskussion:Aharonov-Bohm-Effekt

aus Wikipedia, der freien Enzyklopädie
Letzter Kommentar: vor 2 Jahren von Ernsts in Abschnitt mit Gravitation
Zur Navigation springen Zur Suche springen

Leider ist fast alles, was hier steht falsch. Für eine bessere Darstellung siehe en:Aharonov-Bohm effect. --Pjacobi 14:28, 11. Jul 2005 (CEST)

Überholt! - Benutzer 87.160.114.85 10:13, 16. Okt. 2007 (CEST)Beantworten

In der englischen Wikipedia steht inhaltlich das Gleiche, nur ausführlicher. 213.7.181.92 21:41, 12. Jul 2005 (CEST)

Auch überholt! - Benutzer 87.160.114.85 10:13, 16. Okt. 2007 (CEST)Beantworten

Bitte vergleichen:
Was ist das Interessante:

  • (en - richtig) is a quantum mechanical phenomenon by which a charged particle is affected by electromagnetic fields in regions from which the particle is excluded
  • (de - falsch) ist ein quantenmechanisches Phänomen, das die physikalische Bedeutung des Vektorpotentials aufzeigt

Kann das Vektorpotential durch den AB-Effekt gemessen werden? Bzw. "mehr" (d.h. Komponenten, die keinen Beitrag zu den Felder liefern) gemessen werden?

  • (en - richtig) Nein: a phase difference Δφ between any two paths with the same endpoints therefore determined by the magnetic flux Φ through the area between the paths (via Stokes theorem
  • (de - unklar und widersprüchlich)
    • Das Vektorpotential A ist nicht (direkt) messbar.
    • Es wirkt sich aber auf das Verhalten des geladenen Teilchens aus, selbst wenn es mit dem physikalisch messbaren Magnetfeld B nicht in Berührung kommt.

Pjacobi 23:03, 12. Jul 2005 (CEST)


Ich habe mir erlaubt, den Überarbeiten-Baustein erstmal raus zu nehmen, nachdem ich ein paar Korrekturen vorgenommen habe. Die englische Version ist (immer noch) besser und vor allem ausführlicher, ich finde aber nicht, dass hier fast alles falsch war. RS


Nein, es war in der Tat nicht falsch. (Siehe Original-Artikel bei den Weblinks, den ich hinzugefügt habe) Nur ist nicht alles in der Quantenmechanik besonders intuitiv. Probleme bereiten manchem Physiker auch Grössen, die nicht observabel sind. Was auch immer es ist, das die Phasenänderung hervorruft, es ist kein EM-Kraftfeld. Es lässt sich höchstens als Funktion des ursprünglichen EM-Kraftfeldes ausdrücken (Siehe Homepage des Mathematikers Gerhard W. Bruhn), das Kraftfeld selber ist nachweisbar nicht vorhanden. --Tesladome 12:03, 27. Jul 2005 (CEST)

Du setzt ungerechtfertigterweise voraus, dass im Quantensystem "Elektronenstrahl/Doppelschlitz/Magnetfeld" das Geschehen nur auf "klassischen" Wegen stattfindet, und nicht dazwischen. --Pjacobi 22:16, 28. Jul 2005 (CEST)

Was DU, Pjacobi offenbar immer noch nicht verstanden hast ist dass dynamische Potentiale etwas anderes sind als statische Potentiale. Das steht in der Original-Veröffentlichung auch so drin. Bei der Diskussion in der englischen Wikipedia ("Scalar field Theory (Pseudoscience) wurde das auch erwähnt. Eichtheorien helfen da auch nicht weiter. Die "Dritte Interpretation", nämlich die Verwendung von Holonomien als Entitäten ist bei einem 2 Spalt System noch irgendwie verständlich, bei einem 3-Spalt System schon nicht mehr. Der Satz von Stokes ist in DIESEM Fall auch einfach nicht anwendbar, da er auf einem reinen Kräftegleichgewicht "ohne Spannung" aufbaut. Wenn eine Vakuum-Spannung möglich ist, summieren sich gegensätzliche Vektoren nicht einfach zu Null, da potentielle Energie gespeichert wird, die nicht einfach nur Null ist.--Tesladome 19:32, 10. Nov. 2010 (CET)Beantworten


Für etwas Humor zwischendurch: [1] --Pjacobi 22:51, 28. Jul 2005 (CEST)


Huhu!

Die Interpretation des Effektes ist anscheinend nicht ganz eindeutig. Soweit ich weiss, kann man den Effekt entweder als Auswirkung des „nicht beobachtbaren“ Vektorfeldes A deuten oder als Wirkung des B-Feldes. Wegen dem Satz von Stokes lässt sich das zweite rechnerisch durchhalten, die Interpretation ist aber nicht so naheliegend wie bei klassischen Effekten des Elektromagnetismus, weil das B-Feld dann in diesem Fall nichtlokal wirkt. Alle Bahnen bleiben ausserhalb des Zylinders, das B Feld „wirkt“ dort, wo B null ist. In Feynmans Pfadintegral-Formalismus ist m.E. die Erklärung „Auswirkung des Vektorfeldes A“ logischer.

Neuerding betrachtet man alles als „topologischen Effekt“ (im englichen Text vage angedeutet). Das B-Feld ändert die Topologie des Raumes, der für die Elektronen zugänglich ist ...Schwamm drüber. Die jetztige Version scheint mir so weit akzeptabel. RS 29. Jul


Huhu zurück!

Wenn zwei Felder, einersetzs der F-Vierertensor, und andererseits der A-Vierervektor modulo Eichtransformationen, einander eineindeutig bestimmen, kann die Physik nicht eines von Beiden "fundamentaler" beurteilen. Deswegen habe ich übrigens auch den Philosophenlink angefügt, die Philosophie kann natürlich darüber philosophieren.

Ich habe die (vom Stil her sicherlich nicht optimale) Umformulierung durchgeführt, um möglichst deutlich zu machen, dass der Meßwert der letztendlich beim AB-Effekt herauskommt, nichts ist, für das es nicht auch "konventionelle" Messmethoden gäbe.

Pjacobi


Wobei die mathematische Äquivalenz noch nichts darüber aussagt, in welcher Weise das Feld
in die Bewegungsgleichungen eingeht. In Hinblick darauf gibt es tatsächlich - wie ich
nachgeschlagen habe - die Interpretation, dass Potential sei in
der Quantenmechanik "fundamentaler" als das B-Feld (z.B G.Bayms "Lectures on
Quantum Mechanics"). Ich habe den Zusatz etwas umformuliert und hoffe, du bist einverstanden.
Jetzt steht es etwa so wie in Sakurais "Modern Quantum Mechanics". RS


Pjacobi hat nicht ganz unrecht ... auch ich sehe eine Verwandtschaft zum Problem mit der Henne und dem Ei. Einerseits können wegen der Notwendigkeit vom kanonischen Formalismus in der Quantenmechanik die Potentiale nicht aus den Gleichungen eliminiert werden. Andererseits scheinen die Potentiale wegen der Eichinvarianz keine separate Bedeutung zu haben .... Wir sollten halt berichten, welche Interpretationen es gibt und was sich da getan hat seit 1959. --Tesladome 15:04, 1. Aug 2005 (CEST)

Relativphase[Quelltext bearbeiten]

Hallo,
mir fehlt bei dem Artikel irgendwie eine Bemerkung in die Richtung, dass das eigentlich Neue bei diesem Effekt die Tatsache war, dass die Phase der Wellenfunktion plötzlich physikalisch messbare Auswirkungen zeigte und sich nicht im Betragsquadrat weghob, wie es sonst geschieht.
Das ist in meinen Augen ein viel wichtigerer Aspekt beim Aharonov-Bohm-Effekt als die Frage, ob Vektorpotential oder B-Feld.
Was meint ihr, wäre es sinnvoll hier in einer Ergänzung noch eine kleine Herleitung zu zeigen, bei der man diese "Erhaltung" der Relativphase sieht?
Gruß, --Rene 14:32, 8. Aug 2005 (CEST)

Hi, Wenn du den Aspekt Phasenverschiebung näher betrachten willst, wäre es sinnvoll, mal was über die so genannten Berry´s Phasen zu schreiben (z.B. http://www.mi.infm.it/manini/berryphase.html). Mit diesem Begriff werden AB-Effekt und verschiedene andere Phänomene behandelt, die mit F und A gar nichts zu tun haben. RS 10. August 5

@Rene Die Eichinvarianz relativiert den Effekt zu sehr ("ist doch nix Besonderes"). Jede deutliche Hervorhebung der Unterschiede ist willkommen. Meine persönliche Deutung: Feldstärken sind eichinvariant, Potentiale sind eichabhängig. Das mag für den einen oder anderen Physiker zu philosophisch sein, aber man bedenke, dass die aktuelle Interpretation der Quantenmechanik ebenfalls nicht philosophisch neutral ist. --Tesladome 19:59, 8. Aug 2005 (CEST)

@Tesladome Hm, kann dir irgendwie nicht folgen.

Das ist ein weit verbreitetes Problem zwischen Paraphysikern und Physikern (Bin eigentlich Informatiker). Ich sehe manche Sachen etwas lockerer als ein Physiker, kann mich aber nicht immer so ausdrücken, dass man mich versteht.

Inwiefern relativiert die Eichinvarianz den Effekt?

Nun, ich habe das Gefühl, dass der Effekt von allergrösster Bedeutung ist und dass diese Bedeutung üblicherweise nicht erkannt wird (siehe unten). Gerade die Existenz der Eichinvarianz macht es schwierig, das physikalisch zu begründen, d.h. hier muss man fast schon die Philosophie bemühen (weil nicht observable Grössen im Spiel sind).

Und was meinst du mit "Hervorhebung der Unterschiede"?

Ich beziehe mich damit hauptsächlich auf die Interpretationen der Ergebnisse. Gerhard W. Bruhn interpretiert den Effekt so, dass die Elektronen nur das elektrische Feld "fühlen". Es ist aber auch möglich, dass die Potentiale direkt wirken. Beispiel aus der klassischen Mechanik als Analogie: Eine Steinschleuder. Wenn Du sie spannst, hast Du ein Potential. Wenn Du loslässt, wirkt das Gummi ja "direkt" auf den Stein, oder nicht? Die Auffassung, die Potentiale als Realobjekt anzusehen, ist unter Quantenphysikern relativ verbreitet und gestattet eine "Erklärung" des AB-Effekts in Form einer lokalen Kontaktwechselwirkung zwischen Elektronenwellenfunktion und Potential. Eine weitere Lokalitätsannahme, die man in Feldtheorien normalerweise macht, wird auf diese Weise ebenfalls erfüllt: das Nahewirkungsprinzip.

Das Feldstärken eichinvariant sind und Potentiale nicht, ist insofern klar, als dass Potentiale (von ihrer Konstruktion her) eigentlich nur mathematische Hilfsmittel sind und i.A. keine physkalisch messbare Bedeutung haben.

Ok, Potentiale sind schon in der klassischen Theorie nicht eichinvariant. Aber dass sie nur mathematische Hilfsmittel sind, wage ich zu bezweifeln (siehe oben). Die Physik kann zwar (momentan) nicht zu unterscheiden, ob die Potentiale "primär" oder "Effekt" sind, aber es macht genauso wenig Sinn, die Felder als primär zu interpretieren und die Potentiale als Rechenhilfsmittel. (Mir ist übrigens auch keine Formulierung der Quantenmechanik bekannt, die Felder statt Potentiale verwendet.)

Abgesehen von z.B. der elektr. Spannung als Potentialdifferenz oder Temperatur. Allerdings sind mir die oben zitierten Intepretationen des Vektorpotentials nicht geläufig.

Die Interpretationen sollten einen extra Abschnitt bekommen und so gekennzeichnet sein, dass sie sich im Grunde ausserhalb des Zuständigkeitsbereiches der Physik bewegen. Drei Interpretationen gibt es, soviel ich weiss.

Vielleicht liegt es daran, dass ich erst im 5. Semester bin, aber irgendwie finde ich, dass die obige Diskussion über die Bedeutung des Vektorpotentials am Thema vorbei geht, wenigstens aber darüber hinaus schießt.

Ich sehe das etwas ganzheitlicher, jede Wissenschaft, die was dazu zu sagen hat, kann hier erwähnt werden. Die Physiker interpretieren oft sehr einseitig, auch wenn ein Thema kontrovers diskutiert wurde. Beispiel: Quantenmechanik mit verborgenen Variablen (Bohmsche_Mechanik) und ohne verborgene Variablen (Kopenhagener Interpretation etc.). Es soll sogar Physiker geben, die eine Theorie mit verborgenen Variablen ganz abstreiten. Eine korrekte enzyklopädische Behandlung muss hier differenzieren, da die Physik den AB-Effekt nicht "gepachtet" hat.

Habe übrigens eine kleine Ergänzung im Artikel vorgenommen. Und zwar B als Rotation von A geschrieben. Mit einem Link zur Erklärung, was die Rotation mathematisch ist. Für uns ist das trivial, aber der Vollständigkeit wegen schien es mir angebracht.
Grüße, --Rene 22:13, 8. Aug 2005 (CEST)

Ok, ich kümmere mich auch erstmal um eine saubere Darstellung der Physik. --Tesladome 12:42, 10. Aug 2005 (CEST)

Wie gesagt, ich finde die Diskussion "Potential oder Feld" recht müßig, deswegen gehe ich jetzt auch nicht näher darauf ein. Aber wie du schon sagst, als Enzyklopädie sollte wikipedia das zumindest erwähnen - in einem gesonderten Abschnitt.
Das die Physik den AB-Effekt nicht gepachtet hat mag richtig sein (auch wenn ich ihm noch nie woanders begegnet bin). Andererseits ist der AB-Effekt nun einmal aus der Physik. Dementsprechend sollte eine physikalische Erklärung inklusive Interpretation hier den Vorzug erhalten. Insbesondere philosophische Interpretationen gehören in einen gesondert gekennzeichneten Abschnitt, finde ich. Dann weiß der Leser von vornherein, "worauf er sich einlässt".
Wenn du dich darum kümmern magst? Ich habe von den obigen Interpretationen wenig Ahnung und mag mich eigentlich auch nicht damit befassen.
Ich werde mich in den nächsten Tagen mal hinsetzen und etwas zur physikalisch-mathematischen Herleitung des AB-Effekts schreiben. Das stelle ich dann hier auf die Diskussionsseite. Dann können wir entscheiden, ob es hinein passt, ob was "abgespeckt", korrigiert werden muss, oder sonstiges.
Grüße, --Rene 18:03, 13. Aug 2005 (CEST)

elektrischer AB-Effekt[Quelltext bearbeiten]

Ich finde leider nicht mehr das Lehrbuch in dem ich das gelesen habe, aber auch in der englischen Wiki-Version wird der elektrische AB-Effekt aufgeführt. Wenn ich mich recht erinnere funktioniert das Experiment so: Man hat einen Metallischischen Kasten mit Doppelspalt und schickt Elektronenstrahl gepulst drauf. Erst wenn ein Elektronenpaket den Spalt passiert haben, läd man ihn statisch auf. Der Kasten ist ja ein Faraday-Käfig, also dringen keine E-Felder ein, aber das Skalarpotential bewikt dennoch eine Verschiebung des Interferenzmusters. Ich schlage daher erstmal vor, die englische Version einfach zu übersetzen und wenn ich meine Quelle wiederfinde, dann kann ich den experimentellen Aufbau hinzufügen. ----Do ut des 04:03, 29. Aug. 2008 (CEST)Beantworten

Ich habe (wieder) zwei Weblinks gelöscht, da sie aus formalen und technischen Gründen IMHO Wikipedia:Verlinken widersprechen:

  • Für die Betrachtung ist ein ein unübliches Plugin erforderlich
  • Der AB-Effekt gehört seit Jahrzehnten zum Kanon der Quantenmechanik, es ist weder nötig noch sinnvoll "unusual research" zu bemühen.

Pjacobi 17:51, 29. Jul 2005 (CEST)

Dazu Folgendes:

  • bei Wikipedia:Verlinken steht nicht geschrieben, dass "unübliche" Plugins nicht erlaubt sind.
  • Das Plugin ist ermöglicht es, gescannte Artikel aus physikalischen Zeitschriften komprimiert zu übertragen.
  • Du willst den AB-Effekt und die Implikationen nicht wahrhaben und bist daher nicht der richtige Ansprechpartner, um einen Artikel über diesen Effekt bewerten zu können.

--Tesladome 19:07, 29. Jul 2005 (CEST)

Bitte lese meinen obigen Beitrag und Wikipedia:Verlinken jeweils noch einmal. --Pjacobi 19:13, 29. Jul 2005 (CEST)

Ok, hab's gesehen.

  • Keine Links auf Internetseiten mit bevormundenden Mechanismen

Die Umwandlung in PDF war aber einfacher, als ich gedacht hatte. Einfach auf Drucken gehen und den PDF Writer auswählen. Hier sind die Ergebnisse:

Ist das in Ordnung? Wir können auch den Scientific American Artikel weglassen, aber der Originalartikel ist sehr brauchbar, finde ich. --Tesladome 20:51, 29. Jul 2005 (CEST)

Wenn Links direkt auf pdf-Dateien (oder andere Standardformate) führen, ist wohl nichts einzuwenden. Können die pdf-Files bei foquss.de stehen bleiben? 30.7 RS

Ja, können dort stehenbleiben. Aber Peter Jacobi hat was (vorerst) was dagegen. Wie wäre es, wenn wir die wesentlichen Teile direkt in den Artikel schreiben? --Tesladome 15:28, 30. Jul 2005 (CEST)

Jetzt wo ich sehen konnte, was auf den auf den Seiten ist, muss ich erst recht ein Verlinkung widersprechen. Wir dürfen nicht wissentlich auf Seiten mit Verstößen gegen das Urheberrecht verlinken. Oder gibt es ein Erlaubnis von SCiAm und Physical Review? Warum schreibst Du das nicht als Quellenangaben in den ==Literatur== Absatz? --Pjacobi 11:43, 30. Jul 2005 (CEST)

Ok, ich schreibe es vorerst als Quellenangabe rein und frage Physical Review und Scientific American um Erlaubnis, es verlinken zu dürfen. Eine schriftliche Bestätigung sollten wir schon haben, das sehe ich ein. --Tesladome 15:25, 30. Jul 2005 (CEST)

Tut mir leid, aber ich finde die deutsche Version einfach nicht gut und richtig. Fuer mich ist der Punkt bei der Sache nunmal die Phase und nicht der magnetische Fluss, oder was?

Lorentzkraft[Quelltext bearbeiten]

Im Abschnitt "Theorie" ist die Lorentzkraft falsch. Da ist ein "c" zuviel, hab ich ausgebessert (nicht signierter Beitrag von 85.195.69.104 (Diskussion) 11:32, 4. Apr. 2011 (CEST)) Beantworten

Lorentzkraft[Quelltext bearbeiten]

Im Abschnitt "Theorie" ist die Lorentzkraft falsch. Da ist ein "c" zuviel, hab ich ausgebessert - OK, versucht auszubessern... (nicht signierter Beitrag von 85.195.69.104 (Diskussion) 11:32, 4. Apr. 2011 (CEST)) Beantworten

"Pfadintegral"[Quelltext bearbeiten]

Das Wort sollte durch "Wegintegral" ersetzt werden, da es um die Integrale über _einen_ Weg geht, nicht um das Integral über alle möglichen Wege. --62.52.48.68 13:32, 15. Jan. 2013 (CET)Beantworten

Potenziale[Quelltext bearbeiten]

Auch wenn man immer wieder an verschiedenen Stellen gegenteiliges liest: Potenziale sind nicht durch den AB-Effekt messbar. Sie würden ihre Eichfreiheit verlieren, genauso wie die Wellenfunktion ihre Phaseninvarianz verlieren würde. Dieser Fehlschluss folgt meistens aus fehlerhaften Herleitungen (so auch in einigen Lehrbüchern). Fehlerhafte Herleitungen lassen sich meistens folgendermaßen identifizieren: Kommt in der Herleitung eine (meist Weyl-) Eichtransformation vor, sollte man die Eichfunktion einmal explizit ausrechnen. Hat diese eine Singularität (meist in der Mitte der Spule), dann kann ich den Satz von Stokes nicht anwenden. Wird das in der Herleitung trotzdem gemacht, ist das Ergebnis nicht konsistent. --130.149.114.95 15:51, 15. Okt. 2019 (CEST)Beantworten

mit Gravitation[Quelltext bearbeiten]

Siehe https://science.orf.at/stories/3210961/ Aharonov-Bohm-Effekt - Die Schwerkraft spielt nach Quantenregeln

--Ernsts (Diskussion) 19:19, 24. Jan. 2022 (CET)Beantworten